Can n be treated as a constant in this integral?

  • Thread starter Thread starter kingwinner
  • Start date Start date
  • Tags Tags
    Integrate
Click For Summary
In the integral n∫(2^n - t)dt from 0 to n, n can be treated as a constant. This allows for the simplification of the integrand to 2^n * 2^(-t). The anti-derivative of 2^(-t) is 2^(-t)/ln(2), and thus the integral can be evaluated by pulling 2^n out. As long as n is not a function of t, it is permissible to treat it as a constant during integration. This understanding clarifies how to approach integrals where variables appear in both the integrand and limits.
kingwinner
Messages
1,266
Reaction score
0

Homework Statement


n
∫2n-tdt
0

Homework Equations


N/A


The Attempt at a Solution


I've been wondering about the correct way to deal with this type of integral for quite a long time. To me, the above integral looks like something of the form:
n
∫f(n,t)dt
0
n appears in the integrand AND in the limits of integration, how can I integrate in this case?

I am just wondering whether n can be treated as a "constant" in the above integral, i.e. can I treat 2^n as a constant and pull the 2^n OUT of the integral and evaluate
n
∫2-tdt ?
0

Thank you for explaining!
 
Physics news on Phys.org
Yes, the n is a constant in this case.
 
First, 2n-t= 2n 2-t.

Second, the derivative of 2t is (ln 2)2t so the anti-derivative is 2t/ln(2).
 
morphism said:
Yes, the n is a constant in this case.
So even though "n" appears in the integrand and also appears in the limits of integration, we can still treat the "n" in the integrand as a constant and use the property ∫cf(t)dt=c∫f(t)dt ?
 
Last edited:
If you integrating with respect to t, you don't have to worry about anything else unless n is a function of t, which it is not stated to be.
 
Question: A clock's minute hand has length 4 and its hour hand has length 3. What is the distance between the tips at the moment when it is increasing most rapidly?(Putnam Exam Question) Answer: Making assumption that both the hands moves at constant angular velocities, the answer is ## \sqrt{7} .## But don't you think this assumption is somewhat doubtful and wrong?

Similar threads

  • · Replies 5 ·
Replies
5
Views
1K
  • · Replies 2 ·
Replies
2
Views
2K
  • · Replies 9 ·
Replies
9
Views
3K
  • · Replies 10 ·
Replies
10
Views
2K
  • · Replies 4 ·
Replies
4
Views
2K
  • · Replies 14 ·
Replies
14
Views
3K
  • · Replies 16 ·
Replies
16
Views
4K
  • · Replies 3 ·
Replies
3
Views
2K
  • · Replies 9 ·
Replies
9
Views
2K
Replies
11
Views
2K